K
Khách

Hãy nhập câu hỏi của bạn vào đây, nếu là tài khoản VIP, bạn sẽ được ưu tiên trả lời.

8 tháng 5 2018

\(\frac{a^2}{b^3}+\frac{b^2}{c^3}+\frac{c^2}{a^3}=\frac{1}{b}+\frac{1}{c}+\frac{1}{a}\)

=> \(\frac{a^2}{b^3}+\frac{b^2}{c^3}+\frac{c^2}{a^3}=\frac{1}{a}+\frac{1}{b}+\frac{1}{c}\)

20 tháng 5 2018

\(\frac{a^2}{b^3}+\frac{1}{a}+\frac{1}{a}\ge3\cdot\frac{1}{b}\)

22 tháng 3 2019

Áp dụng bất đẳng thức Cô-si ta có:

\(\dfrac{a^2}{b^3}+\dfrac{1}{a}+\dfrac{1}{a}\ge\sqrt[3]{\dfrac{a^2}{b^3}.\dfrac{1}{a}.\dfrac{1}{a}}=\dfrac{3}{b}\)

\(\dfrac{c^2}{a^3}+\dfrac{1}{c}+\dfrac{1}{c}\ge\sqrt[3]{\dfrac{c^2}{a^3}.\dfrac{1}{c}.\dfrac{1}{c}}=\dfrac{3}{a}\)

\(\dfrac{c^2}{a^3}+\dfrac{1}{c}+\dfrac{1}{c}\ge\sqrt[3]{\dfrac{c^2}{a^3}.\dfrac{1}{c}.\dfrac{1}{c}}=\dfrac{3}{a}\)

Cộng theo vế ta được:

\(\dfrac{a^2}{b^3}+\dfrac{b^2}{c^3}+\dfrac{a^2}{a^3}+\dfrac{2}{a}+\dfrac{2}{b}+\dfrac{2}{c}\ge3\left(\dfrac{1}{a}+\dfrac{1}{b}+\dfrac{1}{c}\right)\)

\(\Leftrightarrow\dfrac{a^2}{b^3}+\dfrac{b^2}{c^3}+\dfrac{c^2}{a^3}\ge\dfrac{1}{a}+\dfrac{1}{b}+\dfrac{1}{c}\)

2 tháng 7 2019

Áp dụng Svac

\(\Sigma\frac{a^3}{b+c}=\Sigma\frac{a^4}{ab+ac}\ge\frac{\left(a^2+b^2+c^2\right)^2}{2\left(ab+bc+ca\right)}\ge\frac{\left(a^2+b^2+c^2\right)^2}{2\left(a^2+b^2+c^2\right)}=\frac{1}{2}\left(a^2+b^2+c^2\right)\)

"=" tại a=b=c

4 tháng 7 2019

E thử làm cách khác ạ:))

Không mất tính tổng quát,giả sử \(a\ge b\ge c\)

\(\Rightarrow\hept{\begin{cases}a^2\ge b^2\ge c^2\\\frac{a}{b+c}\ge\frac{b}{a+c}\ge\frac{c}{a+b}\end{cases}}\)

Áp dụng BĐT Trebysev ta có:

\(a^2\cdot\frac{a}{b+c}+b^2\cdot\frac{b}{a+c}+c^2\cdot\frac{c}{a+b}\ge\frac{a^2+b^2+c^2}{3}\cdot\left(\frac{a}{b+c}+\frac{b}{c+a}+\frac{c}{a+b}\right)\)

\(\ge\frac{a^2+b^2+c^2}{3}\cdot\frac{3}{2}\left(nesbitt\right)\)

\(=\frac{a^2+b^2+c^2}{2}\)

Dấu "=" xảy ra khi  \(a=b=c\)


 

9 tháng 8 2019

2) Theo nguyên lí Dirichlet, trong ba số \(a^2-1;b^2-1;c^2-1\) có ít nhất hai số nằm cùng phía với 1.

Giả sử đó là a2 - 1 và b2 - 1. Khi đó \(\left(a^2-1\right)\left(b^2-1\right)\ge0\Leftrightarrow a^2b^2-a^2-b^2+1\ge0\)

\(\Rightarrow a^2b^2+3a^2+3b^2+9\ge4a^2+4b^2+8\)

\(\Rightarrow\left(a^2+3\right)\left(b^2+3\right)\ge4\left(a^2+b^2+2\right)\)

\(\Rightarrow\left(a^2+3\right)\left(b^2+3\right)\left(c^2+3\right)\ge4\left(a^2+b^2+1+1\right)\left(1+1+c^2+1\right)\) (2)

Mà \(4\left[\left(a^2+b^2+1+1\right)\left(1+1+c^2+1\right)\right]\ge4\left(a+b+c+1\right)^2\) (3)(Áp dụng Bunhicopxki và cái ngoặc vuông)

Từ (2) và (3) ta có đpcm.

Sai thì chịu

9 tháng 8 2019

Xí quên bài 2 b:v

b) Không mất tính tổng quát, giả sử \(\left(a^2-\frac{1}{4}\right)\left(b^2-\frac{1}{4}\right)\ge0\)

Suy ra \(a^2b^2-\frac{1}{4}a^2-\frac{1}{4}b^2+\frac{1}{16}\ge0\)

\(\Rightarrow a^2b^2+a^2+b^2+1\ge\frac{5}{4}a^2+\frac{5}{4}b^2+\frac{15}{16}\)

Hay \(\left(a^2+1\right)\left(b^2+1\right)\ge\frac{5}{4}\left(a^2+b^2+\frac{3}{4}\right)\)

Suy ra \(\left(a^2+1\right)\left(b^2+1\right)\left(c^2+1\right)\ge\frac{5}{4}\left(a^2+b^2+\frac{1}{4}+\frac{1}{2}\right)\left(\frac{1}{4}+\frac{1}{4}+c^2+\frac{1}{2}\right)\)

\(\ge\frac{5}{4}\left(\frac{1}{2}a+\frac{1}{2}b+\frac{1}{2}c+\frac{1}{2}\right)^2=\frac{5}{16}\left(a+b+c+1\right)^2\) (Bunhiacopxki) (đpcm)

Đẳng thức xảy ra khi \(a=b=c=\frac{1}{2}\)

28 tháng 7 2019

Đề chơi căng nhỉ?

a) Dễ chứng minh VP =< 3

BĐT \(\Leftrightarrow\left(\frac{a+b}{1+a}-1\right)+\left(\frac{b+c}{1+b}-1\right)+\left(\frac{c+a}{1+c}-1\right)\ge0\)

\(\Leftrightarrow\frac{b-1}{1+a}+\frac{c-1}{1+b}+\frac{a-1}{1+c}\ge0\)

\(\Leftrightarrow\frac{\left(b-1\right)^2}{\left(1+a\right)\left(b-1\right)}+\frac{\left(c-1\right)^2}{\left(1+b\right)\left(c-1\right)}+\frac{\left(a-1\right)^2}{\left(1+c\right)\left(a-1\right)}\) >=0

Áp dụng BĐT Cauchy-Schwarz dạng Engel vào VT ta có đpcm.

P/s: Èo, sao đơn giản thế nhỉ? Em có làm sai chỗ nào chăng?

28 tháng 7 2019

èo, sai rồi:( đẳng thức xảy ra khi a = b = c = 1 nên cái mẫu = 0 do đó vô lí => bài em sai mất rồi:(( hicc

1 tháng 8 2020

Xét \(\frac{a^3}{a^2+ab+b^2}-\frac{b^3}{a^2+ab+b^2}=\frac{\left(a-b\right)\left(a^2+ab+b^2\right)}{a^2+ab+b^2}=a-b\)

Tương tự, ta được: \(\frac{b^3}{b^2+bc+c^2}-\frac{c^3}{b^2+bc+c^2}=b-c\)\(\frac{c^3}{c^2+ca+a^2}-\frac{a^3}{c^2+ca+a^2}=c-a\)

Cộng theo vế của 3 đẳng thức trên, ta được: \(\left(\frac{a^3}{a^2+ab+b^2}+\frac{b^3}{b^2+bc+c^2}+\frac{c^3}{c^2+ca+a^2}\right)\)\(-\left(\frac{b^3}{a^2+ab+b^2}+\frac{c^3}{b^2+bc+c^2}+\frac{a^3}{c^2+ca+a^2}\right)=0\)

\(\Rightarrow\frac{a^3}{a^2+ab+b^2}+\frac{b^3}{b^2+bc+c^2}+\frac{c^3}{c^2+ca+a^2}\)\(=\frac{b^3}{a^2+ab+b^2}+\frac{c^3}{b^2+bc+c^2}+\frac{a^3}{c^2+ca+a^2}\)

Ta đi chứng minh BĐT phụ sau: \(a^2-ab+b^2\ge\frac{1}{3}\left(a^2+ab+b^2\right)\)(*)

Thật vậy: (*)\(\Leftrightarrow\frac{2}{3}\left(a-b\right)^2\ge0\)*đúng*

\(\Rightarrow2LHS=\Sigma_{cyc}\frac{a^3+b^3}{a^2+ab+b^2}=\Sigma_{cyc}\text{ }\frac{\left(a+b\right)\left(a^2-ab+b^2\right)}{a^2+ab+b^2}\)\(\ge\Sigma_{cyc}\text{ }\frac{\frac{1}{3}\left(a+b\right)\left(a^2+ab+b^2\right)}{a^2+ab+b^2}=\frac{1}{3}\text{​​}\Sigma_{cyc}\left[\left(a+b\right)\right]=\frac{2\left(a+b+c\right)}{3}\)

\(\Rightarrow LHS\ge\frac{a+b+c}{3}=RHS\)(Q.E.D)

Đẳng thức xảy ra khi a = b = c

P/S: Có thể dùng BĐT phụ ở câu 3a để chứng minhxD:

27 tháng 7 2020

1) ta chứng minh được \(\Sigma\frac{a^4}{\left(a+b\right)\left(a^2+b^2\right)}=\Sigma\frac{b^4}{\left(a+b\right)\left(a^2+b^2\right)}\)

\(VT=\frac{1}{2}\Sigma\frac{a^4+b^4}{\left(a+b\right)\left(a^2+b^2\right)}\ge\frac{1}{4}\Sigma\frac{a^2+b^2}{a+b}\ge\frac{1}{8}\Sigma\left(a+b\right)=\frac{a+b+c+d}{4}\)

bài 2 xem có ghi nhầm ko

30 tháng 7 2018

\(\frac{1}{a^2}+\frac{1}{b^2}=\frac{a}{a^3}+\frac{1}{b^2}\ge\frac{\left(\sqrt{a}+1\right)^2}{a^3+b^2}\ge\frac{4\sqrt{a}}{a^3+b^2}\)

Cứ tiếp tục như vậy ta sẽ có đpcm. dấu = xảy ra khi a=b=c=1

31 tháng 8 2018

Ta có: Theo bất đẳng thức cauchy schwarz và bất đẳng thức cauchy với a;b;c>0 ta có:

\(\dfrac{1}{a^2}+\dfrac{1}{a^2}=\dfrac{\left(\sqrt{a}\right)^2}{a^3}+\dfrac{1}{a^2}\ge\dfrac{\left(\sqrt{a}+1\right)^2}{a^3+a^2}\ge\dfrac{4\sqrt{a}}{a^3+a^2}\)(1)

Tương tự \(\dfrac{1}{b^2}+\dfrac{1}{b^2}\ge\dfrac{4\sqrt{b}}{b^3+b^2}\left(2\right);\dfrac{1}{c^2}+\dfrac{1}{c^2}\ge\dfrac{4\sqrt{c}}{c^3+c^2}\left(3\right)\)

Cộng từng vế (1) ;(2);(3) vế theo vế rồi chia hai vế cho 2 ta có đpcm

1 tháng 9 2018

Sai rồi nhé

KT lại đi bạn ơi

9 tháng 9 2018

ta có : \(\dfrac{1}{a^2}+\dfrac{1}{b^2}=\dfrac{a}{a^3}+\dfrac{1}{b^2}\ge\dfrac{\left(\sqrt{a}+1\right)^2}{a^3+b^2}=\dfrac{a^2+2\sqrt{a}+1}{a^3+b^2}\ge\dfrac{4\sqrt{a}}{a^3+b^2}\)

làm tương tự ta có : \(\dfrac{1}{b^2}+\dfrac{1}{c^2}\ge\dfrac{4\sqrt{b}}{b^3+c^2}\)\(\dfrac{1}{c^2}+\dfrac{1}{a^2}\ge\dfrac{4\sqrt{c}}{c^3+a^2}\)

cộng quế theo quế \(\Rightarrow\) (đpcm)

9 tháng 9 2018

bạn làm sai rồi

cái dòng đầu tiên í

\(\dfrac{\left(\sqrt{a}+1\right)^2}{a^3+b^2}\ne\dfrac{a^2+2\sqrt{a}+1}{a^3+b^2}\)